Caralis’ profits - CR

This topic has expert replies
User avatar
Newbie | Next Rank: 10 Posts
Posts: 7
Joined: Sun Jan 11, 2009 7:18 am
Location: Pune

Caralis’ profits - CR

by longbyte75 » Wed Jul 13, 2011 8:00 am
Since Arlene Hodges was installed as president of the Caralis corporation, profits have increased by an average of 11 percent per year. During the tenure of her predecessor, the corporation's profits averaged a yearly increase of only 7 percent. Obviously Ms. Hodges' aggressive marketing efforts have caused the acceleration in the growth of Caralis' profits.

Which of the following, if true, would most weaken the conclusion drawn above?

(A)The corporation's new manufacturing plant, constructed in the past year, has resulted in a 15 percent increase in production capacity.

(B)For each year of Ms. Hodges' presidency, the corporation's financial records show an increase in profits over the previous year.

(C)During the tenure of Ms. Hodges' predecessor, the corporation began an advertising campaign aimed at capturing consumers between the ages of 24 and 35.

(D)Since Ms. Hodges became president, the corporation has switched the primary focus of its advertising from print ads to radio and television commercials.

(E)Just before he was replaced, Ms. Hodges' predecessor directed the acquisition of a rival corporation, which has nearly doubled the corporation's yearly revenues.

OA is E.


[spoiler]How does doubling of revenues can cause an increase in profits? There might be huge increase in costs as well.[/spoiler]

User avatar
Legendary Member
Posts: 1101
Joined: Fri Jan 28, 2011 7:26 am
Thanked: 47 times
Followed by:13 members
GMAT Score:640

by HSPA » Wed Jul 13, 2011 8:16 am
Conclusion says that profit is due to Mr.Hodge's efforts.

To weaken this we just need to show that something other is the cause for profit.
First take: 640 (50M, 27V) - RC needs 300% improvement
Second take: coming soon..
Regards,
HSPA.

User avatar
Newbie | Next Rank: 10 Posts
Posts: 7
Joined: Sun Jan 11, 2009 7:18 am
Location: Pune

by longbyte75 » Wed Jul 13, 2011 8:29 am
HSPA wrote:Conclusion says that profit is due to Mr.Hodge's efforts.

To weaken this we just need to show that something other is the cause for profit.
Yes exactly. Option C can also be an alternative cause.

Does it necessarily mean that profits have increased due to increase in revenues?

C - will fail to be an alternative cause if the Ad campaign is a failure.
E - will fail to be an alternative cause if the fixed cost increases so much that profits do not rise.

User avatar
Legendary Member
Posts: 1101
Joined: Fri Jan 28, 2011 7:26 am
Thanked: 47 times
Followed by:13 members
GMAT Score:640

by HSPA » Wed Jul 13, 2011 8:40 am
C is out of scope, as attracting a given segment in market is not weakeningthe statement
longbyte75 wrote:
HSPA wrote:Conclusion says that profit is due to Mr.Hodge's efforts.

To weaken this we just need to show that something other is the cause for profit.
Yes exactly. Option C can also be an alternative cause.

Does it necessarily mean that profits have increased due to increase in revenues?

C - will fail to be an alternative cause if the Ad campaign is a failure.
E - will fail to be an alternative cause if the fixed cost increases so much that profits do not rise.
First take: 640 (50M, 27V) - RC needs 300% improvement
Second take: coming soon..
Regards,
HSPA.

User avatar
Legendary Member
Posts: 504
Joined: Tue Apr 19, 2011 1:40 pm
Thanked: 114 times
Followed by:11 members

by knight247 » Wed Jul 13, 2011 9:03 am
(A)Doesn't say enough. An increase in production capacity by 15% is not the strongest indicator of an increase in profits
(B)If anything, it strengthens the arguments
(C)Because Hodges' predecessor started a campaign to attract a particular demographic does not in any way mean that Hodges' marketing campaigns are futile. We are trying to find a solid alternate cause for the company's profits and this statement is open to interpretation.
(D)This one may slightly strengthen the argument
How does doubling of revenues can cause an increase in profits? There might be huge increase in costs as well.
Yes, ur argument does make sense. But which of the other four answer choices is as strong as E? Remember, the answer choice does not have to make 100% sense in real life. It could also be the one which is the least senseless when compared to real life.lol. Hope this helps

Junior | Next Rank: 30 Posts
Posts: 18
Joined: Sun Jul 10, 2011 1:13 pm
Thanked: 1 times

by abhicoolmax » Wed Aug 31, 2011 10:32 am
knight247 wrote:(A)Doesn't say enough. An increase in production capacity by 15% is not the strongest indicator of an increase in profits
(B)If anything, it strengthens the arguments
(C)Because Hodges' predecessor started a campaign to attract a particular demographic does not in any way mean that Hodges' marketing campaigns are futile. We are trying to find a solid alternate cause for the company's profits and this statement is open to interpretation.
(D)This one may slightly strengthen the argument
How does doubling of revenues can cause an increase in profits? There might be huge increase in costs as well.
Yes, ur argument does make sense. But which of the other four answer choices is as strong as E? Remember, the answer choice does not have to make 100% sense in real life. It could also be the one which is the least senseless when compared to real life.lol. Hope this helps
Problems like this and, more importantly, official solutions like this makes me NOT trust anything BUT OGs. I think Kaplan got the solution wrong. Weaken MUST shatter the argument or at-least make the conclusion less believable. Kaplan is making a serious assumption to make ANY correlation b/w profit and revenue - without knowing the cost, nothing can be said. That's my take on this stupid problem.

User avatar
Legendary Member
Posts: 582
Joined: Tue Mar 08, 2011 12:48 am
Thanked: 61 times
Followed by:6 members
GMAT Score:740

by force5 » Wed Aug 31, 2011 2:46 pm
E seems to be the best in the lot. but yes it has holes too.

User avatar
GMAT Instructor
Posts: 613
Joined: Thu Mar 22, 2007 6:17 am
Location: madrid
Thanked: 171 times
Followed by:64 members
GMAT Score:790

by kevincanspain » Wed Aug 31, 2011 3:56 pm
If you think that evidence that weakens an argument must shatter it, you may well find on occasion that none of the choices satisfy you. Really, doesn't E make you question the line of reasoning at all? Acquiring a rival corporation, i.e. eliminating a competitor, could very well allow profit margins to at least remain stable
Kevin Armstrong
GMAT Instructor
Gmatclasses
Madrid

User avatar
GMAT Instructor
Posts: 613
Joined: Thu Mar 22, 2007 6:17 am
Location: madrid
Thanked: 171 times
Followed by:64 members
GMAT Score:790

by kevincanspain » Wed Aug 31, 2011 4:02 pm
kevincanspain wrote:If you think that evidence that weakens an argument must shatter it, you may well find on occasion that none of the choices satisfy you. Really, doesn't E make you question the line of reasoning at all? Acquiring a rival corporation, i.e. eliminating a competitor, could very well allow profit margins to at least remain stable
. It is quite possible Hodge is reaping the benefits of a takeover orchestrated by her predecessor. In fact any major change in the market or among the competition would cast doubt on the argument.
Kevin Armstrong
GMAT Instructor
Gmatclasses
Madrid

Junior | Next Rank: 30 Posts
Posts: 18
Joined: Sun Jul 10, 2011 1:13 pm
Thanked: 1 times

by abhicoolmax » Fri Sep 02, 2011 3:11 am
kevincanspain wrote:
kevincanspain wrote:If you think that evidence that weakens an argument must shatter it, you may well find on occasion that none of the choices satisfy you. Really, doesn't E make you question the line of reasoning at all? Acquiring a rival corporation, i.e. eliminating a competitor, could very well allow profit margins to at least remain stable
. It is quite possible Hodge is reaping the benefits of a takeover orchestrated by her predecessor. In fact any major change in the market or among the competition would cast doubt on the argument.
Thanks Kevin. I was thinking too formally. You are right as long as the choice pokes a hole in the argument and somehow generates enough doubt in asserting the conclusion, it is golden! I will try to change my strategy towards weaken/strengthen accordingly. Thanks :)

Legendary Member
Posts: 2330
Joined: Fri Jan 15, 2010 5:14 am
Thanked: 56 times
Followed by:26 members

by mundasingh123 » Fri Sep 02, 2011 7:39 am
kevincanspain wrote:
kevincanspain wrote:If you think that evidence that weakens an argument must shatter it, you may well find on occasion that none of the choices satisfy you. Really, doesn't E make you question the line of reasoning at all? Acquiring a rival corporation, i.e. eliminating a competitor, could very well allow profit margins to at least remain stable
. It is quite possible Hodge is reaping the benefits of a takeover orchestrated by her predecessor. In fact any major change in the market or among the competition would cast doubt on the argument.
hi kevin could you also state why C doesnt weaken the argument .
I Seek Explanations Not Answers

Legendary Member
Posts: 608
Joined: Sun Jun 19, 2011 11:16 am
Thanked: 37 times
Followed by:8 members

by saketk » Fri Sep 02, 2011 12:47 pm
mundasingh123 wrote:
kevincanspain wrote:
kevincanspain wrote:If you think that evidence that weakens an argument must shatter it, you may well find on occasion that none of the choices satisfy you. Really, doesn't E make you question the line of reasoning at all? Acquiring a rival corporation, i.e. eliminating a competitor, could very well allow profit margins to at least remain stable
. It is quite possible Hodge is reaping the benefits of a takeover orchestrated by her predecessor. In fact any major change in the market or among the competition would cast doubt on the argument.
hi kevin could you also state why C doesnt weaken the argument .
Option C says: During the tenure of Ms. Hodges' predecessor, the corporation began an advertising campaign aimed at capturing consumers between the ages of 24 and 35.

We cannot assume that the advertising campaign was successful. This option only gives us information about the strategy adopted. It does not give us any further information. Hence, it is not weakening the argument.

Legendary Member
Posts: 627
Joined: Thu Jun 23, 2011 9:12 am
Thanked: 4 times
Followed by:1 members

by mankey » Tue Oct 11, 2011 11:15 am
Can someone please help on this one?

Thanks
Mankey